1
$\begingroup$

$\newcommand\F{\mathcal{F}}\newcommand\G{\mathcal{G}}$Suppose $\F$ is a subsheaf with generic rank $0$ of a coherent sheaf $\G$ with generic rank $p$ on a smooth variety $X$. Is there a nonzero map from $\det(\F) \to \bigwedge^n \G$ for some $n$?

$\endgroup$
4
  • 1
    $\begingroup$ I am confused. Doesn't $\newcommand{\F}{\mathscr{F}}\newcommand{\rk}{\mathrm{rk}}\rk(\F)=0$ mean $\F=0$ and in particular, $\det(\F)=\F=0$? $\endgroup$ Jul 25, 2012 at 11:12
  • 1
    $\begingroup$ What do you mean by exterior powers of $\mathcal G$? $\endgroup$
    – diverietti
    Jul 25, 2012 at 12:39
  • 4
    $\begingroup$ Dear john, I think it would help if you titled the question a little more specifically. $\endgroup$ Jul 25, 2012 at 13:07
  • 1
    $\begingroup$ John: I think you should look at the article The projectivity of the moduli space of stable curves. I. Preliminaries on "det'' and "Div''. of Knudsen and Mumford. $\endgroup$ Jul 25, 2012 at 13:18

1 Answer 1

1
$\begingroup$

Yes, assuming that $det(\mathcal F)$ refers to some nonzero $\wedge^k\mathcal F$, presumably the last nonzero one, and an exterior power of $\mathcal G$ is $\wedge^k \mathcal G$.

The map $\mathcal F \to \mathcal G$ naturally induces a map $\wedge^k \mathcal F \to \wedge^k \mathcal G$. This map is always injective, since we can just check this on the fiber, which is a vector space, and that's true for vector spaces. Since $det(\mathcal F)=\wedge^k(\mathcal F)$ is nonzero, and the map to $\wedge^k \mathcal G$ is injective, it must be nonzero.

$\endgroup$
1
  • $\begingroup$ "This map is always injective, since we can just check this on the fiber ..." What precisely do you mean by this? For an injective homomorphism of $\mathcal{O}_X$-modules, the base change to the residue field of a point need not be injective. If the homomorphism is locally split, then I agree with your argument. In fact, I feel that the question of the OP is ill-posed, since he does not tell us what definition of "det" he is using (my guess is the one used by Mumford). $\endgroup$ Jul 26, 2012 at 14:12

Your Answer

By clicking “Post Your Answer”, you agree to our terms of service and acknowledge you have read our privacy policy.

Not the answer you're looking for? Browse other questions tagged or ask your own question.